Đến nội dung

Stranger411 nội dung

Có 85 mục bởi Stranger411 (Tìm giới hạn từ 07-06-2020)



Sắp theo                Sắp xếp  

#393205 $c \le \frac{1}{4n}$

Đã gửi bởi Stranger411 on 04-02-2013 - 21:12 trong Dãy số - Giới hạn

hình như chỗ

hình như chỗ này có vấn đề

Đúng rồi bạn à :)
có $a_n =0$ mới có thể suy ra được công thức truy hồi giữa $a_k$ và $a_{k+1}$

ps: nói thế là gần hết bài toán rồi @@!



#392817 $c \le \frac{1}{4n}$

Đã gửi bởi Stranger411 on 03-02-2013 - 15:17 trong Dãy số - Giới hạn

Cho dãy số $a_1, a_2, ..., a_n$ và số $c$ thỏa mãn 2 điều kiện:
1) $a_n =0$
2) $a_k = c+ \sum_{i=k}^{n-1}(a_i + a_{i+1})$ $\forall k=\overline{0,n-1}$.
Chứng minh $c \le \frac{1}{4n}$



#381518 $p \in A$

Đã gửi bởi Stranger411 on 29-12-2012 - 11:15 trong Số học

Cho tập $A=\left \{ x|x=a^2+2b^2 , a,b\in \mathbb{Z},b \neq 0\right \}$ và 1 số nguyên tố $p$.
Chứng minh rằng nếu $p^2 \in A$ thì $p \in A$



#368417 $\binom{kn}{n}\vdots k^n$

Đã gửi bởi Stranger411 on 10-11-2012 - 15:31 trong Số học

Tìm tất cả số nguyên $n>1$ sao cho:
a) $\binom{3n}{n}\vdots 3^n$
b) $\binom{kn}{n}\vdots k^n$ với mọi $k>1$.


ps: bài kiểm tra 1 tiết lớp mình :(
có rất ít người làm được cả 2 câu.



#393747 $\sum\limits_{k=0}^{n}\binom{x+1...

Đã gửi bởi Stranger411 on 06-02-2013 - 13:39 trong Tổ hợp và rời rạc

Chứng minh:
$$\sum\limits_{k=0}^{n}\binom{x+1}{2k+1}\binom{x-2k}{n-k}2^{2k+1}=\binom{2x+2}{2n+1} \forall x \in \mathbb{R}$$



#395098 $\sum\limits_{k=0}^{n}\binom{x+1...

Đã gửi bởi Stranger411 on 09-02-2013 - 00:29 trong Tổ hợp và rời rạc

Ta có định lý B như sau:
\begin{equation}
\sum\limits_k {{n+ak \choose m+bk}} f_k = \left[ {t^m } \right]\left( {1 + t} \right)^n f\left( {t^{ - b} \left( {1 + t} \right)^a } \right) \quad (b<0)
\end{equation}
====================================
Quay lại bài toán.Áp dụng định lý B cho VT, ta có:
$$\begin{array}{rcl} \sum\limits_{k = 0}^n \binom{x - 2k}{n - k}\binom{x + 1}{2k + 1}2^{2k + 1} &=& \left[ {t^n } \right]\left( {1 + t} \right)^x \left[ {\frac{{\left( {1 + 2\sqrt u } \right)^{x + 1} - \left( {1 - 2\sqrt u } \right)^{x + 1} }}{2\sqrt u} \bigg| u = \frac{t}{{\left( {t + 1} \right)^2 }}} \right]\\
&=& \left[ {t^n } \right]\left( {1 + t} \right)^x \left[ {\frac{{\left( {1 + 2\sqrt t + t} \right)^{x + 1} - \left( {1 - 2\sqrt t + t} \right)^{x + 1} }}{{2\dfrac{\sqrt t}{t+1}\left( {1 + t} \right)^{x + 1} }}} \right]\\
&=& \left[ {t^n } \right]\frac{{\left( {1 + \sqrt t } \right)^{2x + 2} - \left( {1 - \sqrt t } \right)^{2x + 2} }}{2\sqrt t}\\
\text{Mà ta có: }& & \\
\dfrac{\left( {1 + \sqrt t } \right)^{2x + 2} - \left( {1 - \sqrt t } \right)^{2x + 2}}{2\sqrt t} &=& \dfrac{1}{2\sqrt t}\left( {\sum\limits_{k = 0}^\infty {\binom{2x + 2}{k}\left( {\sqrt t } \right)^k } - \sum\limits_{k = 0}^\infty {\binom{2x + 2}{k}\left( { - \sqrt t } \right)^k } } \right)\\
&=& \dfrac{1}{2\sqrt t}.2\sum\limits_{k = 0}^\infty {\binom{2x + 2}{2k + 1}\left( {\sqrt t } \right)^{2k + 1} }\\
&=& \sum\limits_{k = 0}^\infty {\binom{2x + 2}{ 2k + 1}t^k }\\
\Rightarrow \left[ {t^n } \right]\dfrac{\left( {1 + \sqrt t } \right)^{2x + 2} - \left( {1 - \sqrt t } \right)^{2x + 2}}{2\sqrt t } &=& \binom{2x + 2}{2n + 1}\\
\Rightarrow \sum\limits_{k = 0}^n {\binom{x - 2k}{n - k}\binom{x + 1}{2k + 1}2^{2k + 1}} &=&\binom{2x + 2}{2n + 1}
\end{array}$$

Em cũng ko ngờ là lại có một lời giải như thế này.
Đây là bài toán được sáng tạo từ công thức "Leibniz suy rộng".
Không biết cái công thức trên thì khó mà xây dựng cái hàm sinh cho bài này.

Xin trình bày lời giải của 1 chú lớp 11 cùng trường vs Perfectstrong ;)
Xét hàm: $$f(y)=\sum\limits_{k=0}^{n} \binom{x+1}{t}2^ty^t(1+y^2)^{x+1-t},\left | y \right | < 1$$
Bằng cách thay $t=2k+1$, ta được hệ số của $y^{2n+1}$ là:
$$\sum\limits_{k=0}^{n}\binom{x+1}{2k+1}\binom{x-2k}{n-k}2^{2k+1}$$
Mà $f(y)= (2y+ (1+y^2))^{x+1} = (1+y)^{2x+2}$, nên hệ số của $y^{2n+1}$ trong khai triển này là: $\binom{2x+2}{2n+1} $
Q.E.D

@supermember: Anh chưa xem kĩ nhưng lời giải này không biết có đúng không nha :), cái $x$ trong đề không ghi nhưng nó là số thực chứ không phải số tự nhiên, như vậy thì cái công thức nhị thức đưa ra trong lời giải này hơi hên xui àh nha :)

@Stranger411: Theo định nghĩa hệ số nhị thức mở rộng thì
Với u là số thực, k là số nguyên không âm thì

\[\left( \begin{array}{l}
u\\
k
\end{array} \right) = \left\{ \begin{array}{l}
\frac{{u(u - 1) \ldots (u - k + 1)}}{{k!}}(k > 0)\\
1(k = 0)
\end{array} \right.\]
Nên em nghĩ lời giải trên chắc không có vấn đề.



#411157 $a^7 + 7 = b^2$

Đã gửi bởi Stranger411 on 07-04-2013 - 21:14 trong Số học

Tìm tất cả các cặp số nguyên dương $(a,b)$ thỏa mãn:
$$a^7+7=b^2$$




#411014 $$\sum^n_{k=0}2^k \binom{n}{k...

Đã gửi bởi Stranger411 on 07-04-2013 - 12:06 trong Các dạng toán khác

hứng minh:
$$\sum^n_{k=0}2^k \binom{n}{k} \binom{n-k}{\left [ \frac{n-k}{2} \right ]}=\binom{2n}{n}$$

 

@Dark templar: Bài này thấy quen quá,hình như đã có trong ĐTTH,phần Hàm sinh...


Do cái vế phải hình như bị sai thôi anh dark templar à zz


Đảo chiều 1 phát, ta được:

$$\sum\limits_{k = 0}^n {{2^k}\binom{n}{k}\binom{n - k}{\left\lfloor {\frac{{n - k}}{2}} \right\rfloor }}  = \sum\limits_{k = 0}^n {{2^{n - k}}\binom{n}{k}\binom{k}{\left\lfloor {\frac{k}{2}} \right\rfloor }} $$

Vì $\binom{k}{\left\lfloor \frac{k}{2} \right\rfloor}$ là hạng tử tự do trong khai triển $(1+x)\left(x+\frac{1}{x} \right)^{k}$.
Từ đó, ta có:

$$\sum^n_{k=0}2^{n-k} \binom{n}{k} (1+x)\left ( x+ \frac{1}{x} \right )^k = (1+x)\left ( 2+x+ \frac{1}{x} \right )^n= \frac{1}{x^n} (1+x)^{2n+1}$$
Xét hạng tử tự do ở 2 vế, ta được:

$$\sum^n_{k=0}2^k \binom{n}{k} \binom{n-k}{\left [ \frac{n-k}{2} \right ]}=\binom{2n+1}{n}$$



#410910 CMR luôn tồn tại hai số $k$ khác $m$ sao cho: $|a_m-...

Đã gửi bởi Stranger411 on 06-04-2013 - 22:08 trong Tổ hợp và rời rạc

Cho các số dương $a_1;a_2;...;a_7$, $b_1;b_2;...;b_7$ thỏa: $a_i+b_i \le 2$. CMR luôn tồn tại hai số $k$ khác $m$ sao cho: $$|a_m-a_k|+|b_m-b_k| \le 1$$.

 

@Reddevil: Dùng ý tưởng hình học cho mấy bài này thường không được tự nhiên cho lắm zz 

Xét giao của các tập sau với tập $\{(x,y)| x\ge 0, y\ge 0, x+y \leq 2\}$:
$|x-\frac{1}{2}|+|y| \leq \frac{1}{2}$, 
$|x|+|y-\frac{1}{2}| \leq \frac{1}{2}$,
$|x-\frac{3}{2}|+|y| \leq \frac{1}{2}$,
$|x|+|y-\frac{3}{2}| \leq \frac{1}{2}$,
$|x-\frac{1}{2}|+|y-1| \leq \frac{1}{2}$,
$|x-1|+|y-\frac{1}{2}| \leq \frac{1}{2}$.
6 tập này phủ toàn bộ $\{(x,y)| x\ge 0, y\ge 0, x+y \leq 2\}$. 
Như vậy tồn tại ít nhất một tập chứa hai cặp $(a_i,b_i)$ và $(a_j,b_j)$ nào đó.
Khi đó $|a_i-a_j|+|b_i-b_j|\leq 1$.



#359617 $\frac{{d({n^2})}}{{d(n)...

Đã gửi bởi Stranger411 on 06-10-2012 - 23:04 trong Số học

Với mỗi số nguyên dương n,kí hiệu d(n) là số các ước dương của n.Tìm tất cả số nguyên dương m sao cho tồn tại số nguyên dương thỏa $\frac{{d({n^2})}}{{d(n)}} = m$

Đây là bài 3 của IMO 1998
Các bạn có thể tham khảo một số lời giải khác ở đây:
http://www.artofprob...=124439#p124439



#368914 $\binom{kn}{n}\vdots k^n$

Đã gửi bởi Stranger411 on 12-11-2012 - 08:53 trong Số học

Câu a :
+ $n= 1$ thỏa mãn.
+ $n\geq 2$ qui nạp đơn giản ta đc $\binom{3n}{n}\not\vdots 3^n$
Câu b chắc tương tự.

Bài (a) thực ra có trong THTT nhưng người ta không giải.
Em thử giải rõ ra xem nào :P
Mấy bạn lớp anh cũng nói quy nạp như em mà vào thi có làm ra đâu :P


Gợi ý: Câu (b) khác câu a ở chỗ:
câu (a) quy nạp theo $n$
còn câu (b) quy nạp theo $k$



#340310 Chứng minh: $a = {10^k}$

Đã gửi bởi Stranger411 on 26-07-2012 - 01:01 trong Số học

Cho số nguyên dương $a$ thỏa mãn $S\left( {{a^n} + n} \right) = 1 + S\left( n \right)$ với mọi số tự nhiên $n$ lớn tùy ý.
Chứng minh $a$ là một lũy thừa của $10$.

- Kvant -




#313291 ${{a}^{3}}+{{b}^{3}}+{{c}^{3}}+kabc\ge 3+k$

Đã gửi bởi Stranger411 on 29-04-2012 - 14:32 trong Bất đẳng thức - Cực trị

Mình đang viết 1 chuyên đề về phương pháp tìm hằng số lớn nhất cho bđt.
Mong các bạn góp ý :D

Bài toán 1: Cho các số thực không âm a,b,c thỏa mãn $a+b+c=3$. Tìm hằng số k lớn nhất để bđt sau đúng:

$${{a}^{3}}+{{b}^{3}}+{{c}^{3}}+kabc\ge 3+k$$

Bài toán 2: (Stranger411)Cho các số thực không âm a,b,c thỏa $ab+bc+ca=3$. Tìm hằng số k lớn nhất để bđt sau đúng:

$${{a}^{3}}+{{b}^{3}}+{{c}^{3}}+kabc\ge 3+k$$

Chú ý: Với bài toán 2,ta có một số kết quả:

$k=6$: Hệ quả của bđt Schur
$k=7$: Bài toán đã được arqady giải ở Mathlinks
Tuy nhiên,$k=7$ không phải hằng số tốt nhất cho bđt trên.



#312383 CMR: $xy+yz+zx\leq 8$

Đã gửi bởi Stranger411 on 24-04-2012 - 10:36 trong Bất đẳng thức và cực trị

Cho $x,y,z$ la nghiệm của hệ pt:$\left\{\begin{matrix} x^2+xy+y^2=3\\ y^2+yz+z^2=16\end{matrix}\right.$
CMR: $xy+yz+zx\leq 8$

Áp dụng bất đẳng thức Cauchy-Schwarz, ta có:
${{\left( xy+yz+zx \right)}^{2}}$
$=\left[ x\left( y+\frac{z}{2} \right)+z\left( y+\frac{x}{2} \right) \right]$
$\le \left[ {{x}^{2}}+\frac{4}{3}{{\left( y+\frac{x}{2} \right)}^{2}} \right]\left[ {{\left( y+\frac{z}{2} \right)}^{2}}+\frac{3}{4}{{z}^{2}} \right]$
$=\frac{4}{3}\left( {{x}^{2}}+xy+{{y}^{2}} \right)\left( {{y}^{2}}+yz+{{z}^{2}} \right)=64$
$\Leftrightarrow xy+yz+zx\le 8$ (đpcm)



#359606 Tìm nghiệm nguyên

Đã gửi bởi Stranger411 on 06-10-2012 - 22:43 trong Số học

Tìm $p,q\in \mathbb{P}$ thỏa mãn $3pq\mid a^{3pq}-a$ với mọi $a\in \mathbb{Z}^+$

Nếu biết giới hạn $p,q$ và chọn $a$ là căn nguyên thủy của $n$ ngay từ đầu thì bài toán sẽ gọn hơn rất nhiều.
Giả sử $p \ge q$

+ Cho $a=3$, ta có:
${3^{pq}} \equiv 3\left( {\bmod 3pq} \right) \Rightarrow 3\left( {{3^{pq - 1}} - 1} \right) \vdots 3pq \Rightarrow p,q > 3$

+ Cho ${a^{\varphi \left( n \right)}} \equiv 1\left( {\bmod n} \right)$ ( $a$ là căn nguyên thủy của $n$)
Theo định lí Fermat nhỏ: ${a^{p - 1}} \equiv 1\left( {\bmod p} \right)$
Vì ${a^{3pq - 1}} \equiv 1\left( {\bmod p} \right) \Rightarrow p - 1|3pq - 1 \Rightarrow p - 1|3q - 1$
Chứng minh tương tự: $q - 1|3p - 1$
Vì $p \ge q$ nên $3q - 1 \in \left\{ {p - 1;2\left( {p - 1} \right);3\left( {p - 1} \right)} \right\}$
Thay vào điều kiện bài toán, ta được: $\boxed{(p,q)=(11,17),(17,11)}$



#340870 Chứng minh: $a = {10^k}$

Đã gửi bởi Stranger411 on 27-07-2012 - 20:43 trong Số học

Cho n=1.Ta có S(a+1)=2
Suy ra a+1 có dạng:\[a + 1 = 2 \times {10^k}\] hoặc \[a + 1 = {10^k} + {10^h}(k > h)\]

Đến đây có thể dùng 2 tính chất quan trọng của $S(n)$ để giải bài toán.
là $S(m)+S(n) \ge S(m+n)$ và $S(m)S(n) \ge S(mn)$



#347133 Tìm số nguyên $n> 1$ sao cho $\frac{2^{n...

Đã gửi bởi Stranger411 on 16-08-2012 - 10:06 trong Số học

Bài trên lâu lắm rồi :D Mình mở rộng bằng căn nguyên thủy tí nữa cho nó mạnh :D

Mở rộng: Tìm tất cả các số nguyên $n>1$ sao cho tồn tại duy nhất số nguyên $a$ với $0 < a < n!$ sao cho:
\[n!|{a^n} + 1\]

BÀI TOÁN: Xác định tất cả các số nguyên $n> 1$ sao cho $\frac{2^{n}+1}{n^{2}}$ là một số nguyên.

Bài này còn 2 cách giải nữa bằng căn nguyên thủy và LTE ;)



#312448 $\frac{1}{2-\cos A}+\frac{1}{2-\cos B}+\frac{1}...

Đã gửi bởi Stranger411 on 24-04-2012 - 19:17 trong Bất đẳng thức - Cực trị

Hình đã gửiÔng bạn của mình đây mà
Trước hết ta sẽ chứng minh $$cosA+cosB+cosC\geq -\frac{3}{2}$$
Cho $\overrightarrow{OA};\overrightarrow{OB};\overrightarrow{OC}$ là 3 véc tơ đơn vị. ( các góc $A,B,C \in (0; \pi)$)
Ta dễ dàng chứng minh được $(\overrightarrow{OA}+\overrightarrow{OB}+\overrightarrow{OC})^2=3+2(cosA+cosB+cosC) \ge 0$
Từ đây suy ra $$cosA+cosB+cosC\geq \frac{-3}{2}$$
Áp dụng bất đẳng thức AM-HM ta có:
$$\frac{1}{2-cosA}+\frac{1}{2-cosB}+\frac{1}{2-cosC}\geq \frac{9}{3-cosA-cosB-cosC}\geq \frac{9}{3+\frac{3}{2}}=2$$

Một lỗi sai khá cơ bản đó ông bạn Hình đã gửi
$$2\cos (\overrightarrow{OA},\overrightarrow{OB})= \cos2C$$
Từ đó, ta có:
$$cos2A+cos2B+cos2C\geq \frac{-3}{2}$$
Bài này không đơn giản như vậy đâu



#312379 $\frac{1}{2-\cos A}+\frac{1}{2-\cos B}+\frac{1}...

Đã gửi bởi Stranger411 on 24-04-2012 - 10:28 trong Bất đẳng thức - Cực trị

Bài 1: Cho $\Delta ABC$. Chứng minh:
$$\frac{1}{2-\cos A}+\frac{1}{2-\cos B}+\frac{1}{2-\cos C}\ge 2$$


Bài 2: Cho $\Delta ABC$. Chứng minh:
$$\frac{\left( 1-\sin \frac{A}{2} \right)\left( 1+\cos \frac{A}{2} \right)}{\sin \frac{A}{2}\left( 1+\sin \frac{A}{2} \right)}+\frac{\left( 1-\sin \frac{B}{2} \right)\left( 1+\cos \frac{B}{2} \right)}{\sin \frac{B}{2}\left( 1+\sin \frac{B}{2} \right)}+\frac{\left( 1-\sin \frac{C}{2} \right)\left( 1+\cos \frac{C}{2} \right)}{\sin \frac{C}{2}\left( 1+\sin \frac{C}{2} \right)}\ge 2+\sqrt{3}$$



#409245 Tìm số dư của phép chia $S=\prod^{p}_{t=1}(t^...

Đã gửi bởi Stranger411 on 30-03-2013 - 22:03 trong Số học

Bài toán:

Ch0 số nguyên tố lẻ $p=mk+2$ tr0ng đó $m.k\in \mathbb{N}^{*},m>2$. Tìm số dư của phép chia $S=\prod^{p}_{t=1}(t^{m-1}+t^{m-2}+...+t+1)$ ch0 $p$.

Một bài khó đánh giá hơn ;)

Cho số nguyên tố $p \equiv 1( \bmod m)$, $m  >2$. Chứng minh:
\[\prod\limits_{t = 1}^p {\left( {{t^{m - 1}} + {t^{m - 2}} +  \ldots  + 1} \right)}  \equiv 0(\bmod p)\]




#400670 Tô màu các số tự nhiên

Đã gửi bởi Stranger411 on 28-02-2013 - 14:40 trong Tổ hợp và rời rạc

Nếu $n$ là số lẻ thì hoàn toàn thực hiện được. Tô tất cả số chẵn cùng màu đỏ, số lẻ cùng màu xanh. Khi đó, cách tô trên là thỏa vì:
(i) Mỗi số được tô bởi đúng 1 màu và có vô hạn lần tô mỗi màu (vì có vô hạn số lẻ, số chẵn)
(ii) Tổng $n$ số cùng màu là một số cùng màu vì tổng $n$ số lẻ là một số lẻ (vì $n$ lẻ) và tổng $n$ số chẵn là một số chẵn.
======================================
Nhưng nếu $n$ chẵn thì "có thể" không tô được. Nhưng không biết chứng minh sao :))

Cái trường hợp $n$ chẳn của bạn chỉ là điều kiện cần thôi :lol:
Bài này dù $n$ chẳn hay $n$ lẻ đều không tô được :)

Spoiler



#347129 Tìm số nguyên $n> 1$ sao cho $\frac{2^{n...

Đã gửi bởi Stranger411 on 16-08-2012 - 09:52 trong Số học

BÀI TOÁN: Xác định tất cả các số nguyên $n> 1$ sao cho $\frac{2^{n}+1}{n^{2}}$ là một số nguyên.

Bài này ko cần phải dùng đến cấp của 1 số đâu :)

Bổ đề 1: Cho các số nguyên $m,n$ và $a>1$. Ta có: $\gcd \left( {{a^m} - 1,{a^n} - 1} \right) = {a^{\gcd \left( {m,n} \right)}} - 1$
Bổ đề 2: Nếu ${3^b}|{2^a} - 1 \Rightarrow {3^{b - 1}}|a$

Lời giải bài toán:
+ Khi $n=1$, bài toán thỏa mãn.
+ Khi $n>1 \Rightarrow n$ lẻ.
Gọi $p$ là ước nguyên tố lẻ nhỏ nhất của $n$ nên $\gcd \left( {p - 1,n} \right) = 1$.
Ta có: $p|{2^n} + 1|{2^{2n}} - 1$
Theo định lí Ferma nhỏ, ta có: $p|{2^{p - 1}} - 1$.
Áp dụng bổ đề 1, ta được: $p|\gcd \left( {{2^{p - 1}} - {{1,2}^{2n}} - 1} \right) = {2^{\gcd \left( {2n,p - 1} \right)}} - 1$
mà $\gcd \left( {2n,p - 1} \right) \leqslant 2 \Rightarrow p|3 \Rightarrow p = 3$
Đặt $n = {3^k}d$. Dùng bổ đề 2, ta có: ${3^{2k}}|{n^2}|{2^{2n}} - 1 \Rightarrow {3^{2k - 1}}|n \Rightarrow k = 1$
(*) Nếu $d>1$. Gọi $q$ là ước nguyên tố nhỏ nhất của $d$ nên $q \ge 5$.
Lập luận tương tự như trên, ta có: $q=7$
Vậy nên $7|n|{2^n} + 1$. Điều này vô lí vì ${2^n} + 1 \equiv 2,3,5(\bmod 7)$.
(*) Nếu $d=1$, ta có: $n=3$.
Vậy $n=1$ và $n=3$ thỏa mãn đề bài.



#385774 VMO 2013 - Bài 1. Hệ phương trình

Đã gửi bởi Stranger411 on 11-01-2013 - 23:00 trong Phương trình - Hệ phương trình - Bất phương trình

Cách khác: (không dùng Minkowski :D)
ĐKXĐ: $x\neq \dfrac{m\pi }{2}$, $y\neq \dfrac{n\pi }{2}$ ($m$, $n\in \mathbb{Z}$), $xy\geq 0$, $x+y\neq 0$.
Ta chứng minh:
$(\sqrt{{{\sin }^{2}}x+\frac{1}{{{\sin }^{2}}x}}+\sqrt{{{\cos }^{2}}y+\frac{1}{{{\cos }^{2}}y}})^2 + (\sqrt{{{\sin }^{2}}y+\frac{1}{{{\sin }^{2}}y}}+\sqrt{{{\cos }^{2}}x+\frac{1}{{{\cos }^{2}}x}})^2 \ge 20$

Áp dụng $\frac{1}{\sin ^2 x} + \frac{1}{\cos ^2 x} \ge 4$ $ \forall x \neq 0$, ta có:
$(\sqrt{{{\sin }^{2}}x+\frac{1}{{{\sin }^{2}}x}}+\sqrt{{{\cos }^{2}}y+\frac{1}{{{\cos }^{2}}y}})^2 + (\sqrt{{{\sin }^{2}}y+\frac{1}{{{\sin }^{2}}y}}+\sqrt{{{\cos }^{2}}x+\frac{1}{{{\cos }^{2}}x}})^2 $
$\ge 10 + 2 \sqrt{{{\sin }^{2}}x+\frac{1}{{{\sin }^{2}}x}}.\sqrt{{{\cos }^{2}}y+\frac{1}{{{\cos }^{2}}y}} + 2 \sqrt{{{\sin }^{2}}y+\frac{1}{{{\sin }^{2}}y}}.\sqrt{{{\cos }^{2}}x+\frac{1}{{{\cos }^{2}}x}}$

Vậy chỉ cần chứng minh:
$ \sqrt{{{\sin }^{2}}x+\frac{1}{{{\sin }^{2}}x}}.\sqrt{{{\cos }^{2}}y+\frac{1}{{{\cos }^{2}}y}} + \sqrt{{{\sin }^{2}}y+\frac{1}{{{\sin }^{2}}y}}.\sqrt{{{\cos }^{2}}x+\frac{1}{{{\cos }^{2}}x}} \ge 5$ $ \forall x \neq 0$
từ đó quay về chứng minh:
$ \left(\sin^2x + \dfrac{1}{\sin^2x}\right)\left(\cos^2x + \dfrac{1}{\cos^2x}\right) \geq \left(\dfrac{5}{2}\right)^2.$và $ \left(\sin^2y + \dfrac{1}{\sin^2y}\right)\left(\cos^2y + \dfrac{1}{\cos^2y}\right) \geq \left(\dfrac{5}{2}\right)^2.$

Sử dụng bất đẳng thức Cauchy-Schwarz và AM-GM ta có
$$ \begin{aligned} \left(\sin^2x + \dfrac{1}{\sin^2x}\right)\left(\cos^2x + \dfrac{1}{\cos^2x}\right) \ & \geq \left(|\sin x\cos x|+\dfrac{1}{|\sin x\cos x|}\right)^2 \\ & =\left(\dfrac{|\sin 2x|}{2}+\dfrac{1}{2|\sin 2x|}+\dfrac{3}{2|\sin 2x|}\right)^2 \\ & \geq \left( 1+\dfrac{3}{2}\right)^2=\left(\dfrac{5}{2}\right)^2.\end{aligned}$$
Ta có (đpcm).
Suy ra: $\tan x = \tan y= \pm 1$ và $x=y$
Từ đó, ta được $ x=y= \dfrac{\pi }{4}+\dfrac{k\pi }{2}$ ($k\in \mathbb{Z}$).



#400613 Tô màu các số tự nhiên

Đã gửi bởi Stranger411 on 28-02-2013 - 09:13 trong Tổ hợp và rời rạc

Cho số tự nhiên $n>1$. Người ta tô màu tất cả các số tự nhiên bằng 2 màu xanh và đỏ thỏa mãn 2 đk:
1) Mỗi số chỉ được tô một màu và mỗi màu được dùng để tô vô hạn số.
2) Tổng của $n$ số phân biệt cùng màu, là một số cũng có màu đó.
Hỏi cách tô trên có thể thực hiện được hay không ?

Spoiler



#344948 CMR$\left( {\frac{{p - 1}}{2...

Đã gửi bởi Stranger411 on 08-08-2012 - 23:38 trong Số học

Bài toán trên còn 1 cách phát biểu khác như sau:
Hình đã gửi